If the rectangle below has an area of 32 sq. Units, what is the area of thr triangle?

Answers

Answer 1

Answer:

16 units

Step-by-step explanation:

the formula of a triangle is hbb/2


Related Questions

Which graph shows the solution to the equation below? log Subscript 3 Baseline (x + 3) = log Subscript 0.3 (x minus 1) On a coordinate plane, 2 curves intersect at (1, 1). One curve curves up and to the right from quadrant 3 into quadrant 1. The other curve curves down from quadrant 1 into quadrant 4. On a coordinate plane, 2 identical curves are shown. One curve starts at y = negative 3, and the other curve starts at y = 1. On a coordinate plane, a curve and a line are shown. On a coordinate plane, a curve and a cubic function are shown. Mark this and return

Answers

Answer:

On a coordinate plane, 2 curves intersect at (1, 1). One curve curves up and to the right from quadrant 3 into quadrant 1. The other curve curves down from quadrant 1 into quadrant 4

Step-by-step explanation:

The first function is given as:

[tex]log_3(x+3)[/tex]

The second function is given as:

[tex]log_{0.3}(x-1)[/tex]

First we graph both the functions.

We can see that one curves up and to the right from quadrant 3 into quadrant 1. This curve is of  [tex]log_3(x+3)[/tex]

The other curve curves down from quadrant 1 into quadrant 3

Both curves interest almost at (1,1)

See the graph attached below

Blue line represents first function

Green line represents second function

The solution lies on the Red line.

Answer:

A on edge2020

Step-by-step explanation:

took the test

Solve this equation. Explain your reasoning.
0.5x + 0.5 = 4.5​

Answers

Answer:

       x = 9

Step-by-step explanation:

        5          9

 (x -  (—— • x)) -  —  = 0  

        10          2

       

Simplify   1/2

     1          9

 (x -  (— • x)) -  —  = 0  

        2          2

                   x = 9

Answer:

x = 9

Step-by-step explanation:

Firstly, you have to balance the equation.

Start by subtracting 0.5 from both sides. Remember, when solving algebraic equations such as this one, you use reverse PEMDAS, or SADMEP.

Now, you have 0.5x = 4.5

Now, for the next step: division. Divide both sides by 0.5, or in other words, multiply by 2.

x = 9

Hope that helps! Please don't hesitate to contact me if you have any questions :)

Francesca and Irvin are painting the walls of their clubhouse. Working alone Francesca can paint the walls in 6 hours. Irvin can complete the same walls in 4 hours if working alone. How long will it take them, working together, to finish painting. Round your answer to the nearest hundredth is necessary.

Answers

Answer:

Step-by-step explanation:

Work finished by Francesca  in one hour = 1/6

Work finished by Irvin in one hour = 1/4

Work finished by Francesca and Irvin in one hour = 1/6 +1/4

                                                             [tex]=\frac{4}{24}+\frac{6}{24}\\\\=\frac{10}{24}\\\\=\frac{5}{12}\\[/tex]

5/12 of the work is completed by both in an hour

Whole work will be completed in 1*12/5 =12/5 hours = 2 2/5 hours

                                                  = 2 hours 24 minutes

In the ordered pair (3, 4), the value of y is 4. The 4 tells us how many places from zero we should go up or down. Will you go up or down from zero?

Answers

Answer:

Up from zero

Step-by-step explanation:

The 4 in the ordered pair is positive, meaning it will be 4 ABOVE zero.

It won't be down from zero because it is not -4

Answer:

You would have to go up from zero.

Step-by-step explanation:

On a coordinate plane, a line goes through (0, negative 3) and (3, negative 2). A point is at (negative 4, 2).
Find the equation of the line parallel to line h that passes through (–4, 2).

y = one-third x + StartFraction 10 Over 3 Endfraction
y = negative one-third x + two-thirds
y = 3x + 14
y = –3x – 10

Answers

Answer:

Correct option: first one -> y = (1/3)x + (10/3)

Step-by-step explanation:

The linear function that represents a line is:

y = ax + b

Where a is the slope and b is the y-intercept.

First we need to find the slope of the line that goes through (0, -3) and (3, -2).

Using both points, we can find the equation of the line:

x = 0 -> y = -3

-3 = a*0 + b

b = -3

x = 3 -> y = -2

-2 = 3a - 3

3a = 1

a = 1/3

The parallel line needs to have the same slope as the line, so we can model the parallel line with the following equation:

y = (1/3)x + b

The parallel line goes through the point (-4, 2), so we have:

x = -4 -> y = 2

2 = (1/3)*(-4) + b

b = 2 + (4/3)

b = 10/3

So the equation of the parallel line is:

y = (1/3)x + (10/3)

Correct option: first one

Answer:

A.) y = one-third x + StartFraction 10 Over 3 Endfraction

Step-by-step explanation:

PLZ HURRY I'M GETTING TIMED
Lianne drew a sphere with a radius of 7 in. She calculated its volume and wrote it below.


StartFraction question mark pi Over 3 EndFraction inches cubed


What value should be multiplied by Pi in the numerator to correctly show the volume of the sphere?

Answers

Answer:

1372

Step-by-step explanation:

Volume of a Sphere[tex]=\frac{4}{3}\pi r^3[/tex]

If radius of the sphere=7 Inches

Volume[tex]=\dfrac{4}{3}\pi *7^3[/tex]

[tex]=\dfrac{1372\pi}{3}$ cubic inch[/tex]

Therefore, 1372 is the value that should be multiplied by Pi in the numerator to correctly show the volume of the sphere.

Answer:

1372

Step-by-step explanation:

just took the quiz

Answer this question to get Brainliest. The answer will be deleted if “troll”. Please answer ASAP

Answers

Answer: 129.81 m, roughly 130 m

Step-by-step explanation: Let me know if you need an explanation.

Answer:

Applying the alternative interior pair of angles theorem => B = 31 deg

Applying the trigonometry property of tangent, the distance from the cliff to the boat is calculated by:

D = 78/tan(B) = 78/tan(31) =~129.81

Hope this helps!

:)

A camera is marked down from $36 to $27. What is the amount of the markdown?

Answers

Answer:

25%

Step-by-step explanation:

36-27 is 9

36*.25=9

therefore the answer is The camera is marked down by 25%.

The graph of f(x)= |x| is transformed to g(x)=|x+1|-7. On which interval is the function decreasing

Answers

.......................

Slope of the line that passes through (3, -11) and (0, 4), please!

Answers

Answer:

-5

Step-by-step explanation:

slope formula= [tex] \frac{y1 - y2}{x1 - x2} [/tex]

slope of line

[tex] = \frac{4 - ( - 11)}{0 - 3} \\ = \frac{4 + 11}{ - 3} \\ = \frac{15}{ - 3} \\ = - 5[/tex]

Which function has a greater slope and y-intercept than the linear function represented in the table?
A) y=2x+8.5
B) y=3x+7.5
C) y=5x+6.5
D) y=10x+5.5

Answers

Answer:

B) y=3x+7.5

Step-by-step explanation:

The equation of the data in the table can be obtained by using any two points.

We will use points (3, 13) and (5,17)

The slope - intercept form is:

y = mx  + b

We find the slope as:

m = (17-13)/(5-3) = 4/2 = 2

The equation becomes:

y = 2x + b

To find the value of b, the y -intercept is:

13 = 2*3 + b ⇒ b= 13 - 6 = 7

So the equation is:

y = 2x + 7

Now let's compare this with the options

A) y = 2x + 8.5

The slope is 2, and y- intercept is 8.5It has same slope and greater y-interceptAnswer is no

B) y=3x+7.5

The slope is 3, and y- intercept is 7.5It has greater slope and greater y-interceptAnswer is yes

C) y=5x+6.5

The slope is 5, and y- intercept is 7.5It has greater slope but smaller y-interceptAnswer is no

D) y=10x+5.5

The slope is 10, and y- intercept is 5.5It has greater slope but smaller y-interceptAnswer is no

Does this graph represent a function? Why or why not?
A. No, because it fails the vertical line test.B. No, because it is not a straight line.
C. Yes, because it passes the horizontal line test.
D. Yes, because it passes the vertical line test.

Answers

Answer:

Step-by-step explanation:

Yes, this graph passes the vertical line test (Answer D)

Find the median 61 61 62 64 66 69 69 70 72 73 74 78

Answers

Answer: the median is 69

Step-by-step explanation:

you organize the numbers from smallest to biggest from there you find the middle and if there is no middle get the two middle numbers and add them from there you divide what you got by 2

Ryan sold a jigsaw puzzle at a yard sale for three dollars and five cents which makes this money amount I’m terms of dollars mark all that apply

Answers

Answer:

$3.05

Step-by-step explanation:

3 dollars = $3

1 cent = 1/100 dollar = $0.01

5 cents = 5 * 1 cent = 5 * $0.01 = $0.05

$3 + $0.05 = $3.05

Please help with corrections ​

Answers

Answer:

7x12x2=168 I think but how did u do this

The right answer is 244 cm^2

please see the attached picture for full solution

hope it helps

Good luck on your assignment

Stay safe....

P=2l+2w. solve for w. show all steps. please

Answers

Answer:

Step-by-step explanation:

You need to get w on one side by itself.

Start out by subtracting 2l from both sides.

p−2l=2w.

Now since w is being multiplied by 2 we need to divided both sides (all terms) by 2.

w=p2−I

Answer:

W=P-L

Step-by-step explanation:

P=2L+2W

Subtract 2L from both sides.

P-2L=2W

Divide both sides by 2

P-L=W

what is the measurement of angle f?

Answers

Answer:

∠ f = 33°

Step-by-step explanation:

Using the sine ratio in the right triangle

sin f = [tex]\frac{opposite}{hypotenuse}[/tex] = [tex]\frac{36}{66}[/tex] , thus

∠ f = [tex]sin^{-1}[/tex] ([tex]\frac{36}{66}[/tex] ) = 33° ( to the nearest degree )

The answer is f equals 33

solve for X will give brainliest!!

Answers

Answer:

4

Step-by-step explanation:

Since ABC is similar to ADE by AA (the triangles share angle A and a right angle), they must have equal proportions. Therefore:

[tex]\dfrac{x}{4}=\dfrac{12}{4+8}[/tex]

Therefore, x=4. Hope this helps!

Answer:

x=4

Step-by-step explanation:

The triangles are similar so we can use ratios to solve

x             4

------ = ----------

12         (4+8)

Using cross products

12x = 4*12

Divide each side by 12

12x/12 = 48/12

x=4

Find the sum, number 8 pleaseeee

Answers

Answer:

i got 20

sorry if this is wrong

Step-by-step explanation:

A triangle was dilated by a scale factor of 6. If sin a° = four fifths and segment DE measures 30 units, how long is segment EF?

triangle DEF in which angle F is a right angle, angle D measures a degrees, and angle E measures b degrees
15.5 units
24 units
30 units
37.5 units

Answers

Answer:

24 units

Step-by-step explanation:

From the description:

angle D measures a degreesopposite to angle D is side EFside DE (which measures 30 units) is the hypotenuse of the triangle.

We also know that

sin(a) = 4/5

From definition:

sin(a) = opposite/hypotenuse

Replacing:

4/5 = EF/DE

EF = (4/5)*DE

EF = (4/5)*30 = 24 units

The length of the segment EF of the triangle DEF is 24 units.

Procedure - Determination of the resulting length after dilating a triangle by a scale factor

In the image attached below we summarize the triangle described in statement, by trigonometric relationship we find that the length of the segment EF is described by the following expression:

[tex]\sin a^{\circ} = \frac{EF}{DE}[/tex] (1)

If we know that [tex]DE = 30[/tex] and [tex]\sin a^{\circ} = \frac{4}{5}[/tex], then the length of the segment EF is:

[tex]EF = DE\cdot \sin a^{\circ}[/tex]

[tex]EF = 30\cdot \left(\frac{4}{5} \right)[/tex]

[tex]EF = 24[/tex]

The length of the segment EF of the triangle DEF is 24 units. [tex]\blacksquare[/tex]

To learn more on right triangles, we kindly invite to check this verified question: https://brainly.com/question/7894175

What is the value of n?

Answers

Answer:

C = 95 degrees

The answer I think would be 95 so C

m∠1 = (4x + 9)° and m∠2 = (x − 14)° in the given figure. Find x. answers: 19 7 8 37

Answers

Answer:

x = 19

Step-by-step explanation:

∠ 1 + ∠ 2 = 90° ( corresponding angle ), substitute values

4x + 9 + x - 14 = 90

5x - 5 = 90 ( add 5 to both sides )

5x = 95 ( divide both sides by 5 )

x = 19

The value of variable x is,

⇒ x = 19

We have to given that;

Measure of angles are,

m∠1 = (4x + 9)°

And, m∠2 = (x − 14)°

Now, We can see that,

Measure of angle 1 and 2 are complementary angle.

Hence, We get;

⇒ m ∠1 + m ∠2 = 90°

Substitute all the values, we get;

⇒ (4x + 9) + (x - 14) = 90

⇒ 5x - 5 = 90

⇒ 5x = 90 + 5

⇒ 5x = 95

⇒ x = 95 / 5

⇒ x = 19

Thus, The value of variable x is,

⇒ x = 19

Learn more about the complementary angle visit;

https://brainly.com/question/20428380

$SPJ6

Is PQR ~ XYZ? If so, name which similarity postulate or theorem applies.

Answers

Answer:

D) cannot be determined

Step-by-step explanation:

We are given that PQR ~ XYZ

Now we are supposed to find which similarity postulate or given theorem applies

1) AA-Similarity : When Two angles of one triangle is equal to its corresponding angles of other triangle.Then, the two triangles are similar by AA Similarity.

2) SSS similarity: When three sides of two triangles are in the same ratio then the two triangles are similar by SSS similarity

3) SAS similarity : When two sides of two triangle are in the same ratio and one angle between two proportional sides of two triangles is congruent then the two triangles are similar by SAS similarity

We do not have enough information to satisfy the above options

So, Option D is true

D) cannot be determined

Answer:Cannot be determined a pex

Step-by-step explanation:

Question
How much time did a person spend at the library over a month for them to spend 40 hours reading a book?
Round your final answer to the nearest whole number.​

Answers

Answer:

The correct answer will be "1.60 hours".

Step-by-step explanation:

Within this test case, let's suppose that the bookstore had been closed on Sundays every other day while there were a total of 5 Sundays throughout the month of thirty days, thus effectively.

Months = 30

Number of sundays = 5

⇒  [tex]Effective \ working \ days = Months - Number \ of \ Sundays[/tex]

⇒                                          [tex]=30-5[/tex]

⇒                                          [tex]=25[/tex]

As we know, the total spend time will be 40 hours, then

⇒  [tex]Time \ spend \ each \ day = \frac{Total \ time \ spens}{Working \ days}[/tex]

On putting the estimated values, we get

⇒                                     [tex]=\frac{40}{30}[/tex]

⇒                                     [tex]=1.60 \ hours[/tex]

Maria mows 2/3 of a lawn in 1/5 hour. How Many similar-sized lawns can she mow in 1 hour?
-must haves:)..
step by step
right answer
makes sense
^thank you and stay safe
^^I will also mark some one the brainiest who ever answers this ;)

Answers

Answer:

10/3 lawn

Step-by-step explanation:

2/3 of a lawn in 1/5 hour

-> 1 hour cab mow = 2/3(lawn) ÷ 1/5(hour)* 1(hour)

= 10/3 = 3,666666667 lawns

Answer:

maria can mow 3 lawns

Step-by-step explanation:

if it takes 1/5 of an hour to mown 2/3 of one lawn

60 divide by .2 = 12

that means it takes 12 miuntes to mown 2/3 of a lawn, so it takes 6 minutes for each 1/3

so 12+6= 18 minutes for one whole lawn

so 18 plus 18 equals 36

so 36 minutes for 2 lawns

then add another 18 minutes and you'll get 54 minutes. That's almost a whole hour

so Maria can mown 3 lawns

( if im wrong, im sorry i tried)

What is the standard form of 5.58 • 10^5?

A. 0.05800

B. 558,000

Answers

It’s 558,000 since the exponent is 5 and that’s means you multiply 5.58x5.58x5.58x5.58x5.58

Answer:

558,000

Step-by-step explanation:

First you multiple 10 by the power of 5 = 10 x 10 x 10 x 10 x 10 = 100000 x 5.58 = 558,000

Using the following image, apply what you have learned about linear pairs and solve for

Answers

Answer:

36

Step-by-step explanation:

I just did it in my head so sry if it’s wrong but I’m trying to give back bc of how much brainless has helped me

1) Find the first three terms of the sequence below. Tn= n^2 + 2n+9

Answers

Answer:

The first 3 terms are 12, 17 and 24.

Step-by-step explanation:

You have to put in n = 1, 2, 3 as they want only first 3 terms :

[tex]Tn = {n}^{2} + 2n + 9[/tex]

[tex]T1 = {1}^{2} + 2(1) + 9 = 12[/tex]

[tex]T2 = {2}^{2} + 2(2) + 9 = 17[/tex]

[tex]T3 = {3}^{2} + 2(3) + 9 = 24[/tex]

PLEASE ANSWER ASAP AND PROVIDE EXPLANATION!

A copy machine is set up to enlarge an original in the ratio 2:3.

What is this enlargement in percent?

Answers

Answer:

It should be about 67%.

Step-by-step explanation:

The ratio 2:3 is the same as the fraction 2/3.

2 divided by 3 is equal to 0.66666...

When it is rounded to the hundredths place, it is 0.67.  

This means that the percentage is 67%.

Answer:

The enlarged object would be 150% larger, but the enlargement itself makes the new picture 50% larger than before as the previous size was already considered 100%.

Step-by-step explanation:

It is much easier to picture this with nice numbers.

Let's say you start with a paper that is size 100 (these numbers have no real meaning they are just for illustrating a point), this would be the 2 in the ratio so the equality would look like: [tex]\frac{2}{3}[/tex]=[tex]\frac{100}{x}[/tex]. At this point you can cross multiply and divide, getting 2x=300. By dividing by 2, you get x=150.

Now percents are out of 100, and you have one of those, only right now it is on the top. All you have to do is flip the fraction, you get [tex]\frac{150}{100}[/tex]=150% is how much it was enlarged when compared to the original, it got 1.5 times larger (you can flip the fraction because you are looking to see how much bigger it got in relation to the starting size).

However, if you are looking for only the enlargement as a percent of the total the answer would be 50% since the new size 150 is 50 larger than 100 and [tex]\frac{50}{100}[/tex]=50%.

Need help with this pls
Show work pls!!!!!

Answers

Answer:

Look at the attachment

Other Questions
Question 10 of 102 PointsWhich of the following is a reason for the growth of the suburbs?A. Racial problems in the citiesOB. Not enough houses in the cities O C. Businesses were leaving the citiesD. All of the aboveSUBMITPREVIOUS 4. Candice leaves on a trip driving at 25 miles per hour. Four hours later, her sisterLiz starts from the same location driving at 50 miles per hour. How long after Lizleaves home will she catch up to Candice?A 9 hoursB 4 hoursC 8 hoursD 7 hours Richard Palm is the accounting clerk of Olive Limited. He uses the source documents such as purchase orders, sales invoices, and suppliers invoices to prepare journal vouchers for general ledger entries. Each day he posts the journal vouchers to the general ledger and the related subsidiary ledgers. At the end of each month, he reconciles the subsidiary accounts to their control accounts in the general ledger to ensure they balance. Discuss the internal control weaknesses and risks associated with the above process. Magnetic reversals _____.cause the movements of tectonic platesprovide strong evidence for sea-floor spreadinghave never occurred during geologic timeconfirm the existence of subduction zones Please Help I will mark as brainliest Bacteria will not be found on fruits or vegetables. TrueFalse write a short note on Labrador Current Which news headline is most clearly biased? A. Support for Senator wens FallingB. Crazed Shoppers Descend on MallC. President Calls Killer Bees "Major Threat"D. Four Planets Align in Rare Event The liver secretes bile, needed to digest fat in our food. The pancreas secretes several enzymes and hormones needed to break down food. Which of the following absorbs digested food? (ASSET)(1 Point)a. Blood cellsb. Small intestinec. Liverd. Stomach Decimal expansion of 5/8 816A company purchased $1,900 of merchandise on July 5 with terms 2/10, n/30. On July 7, it returned $250 worth of merchandise. On July 12, it paid the full amount due. Assuming the company uses a perpetual inventory system, and records purchases using the gross method, the correct journal entry to record the payment on July 12 is:000-12540-18810-397100 how do you separate sand and zinc? if the blue radius below is 4.5 unit lrpendicto the green chord and the segment ab is 4.5 units long, what is the length of the length of the chord? Apply the distributive property to create an equivalent expression. 4(3+1/4c-1/2d)?? A small company that builds wooden fences can currently construct five fences per month for a total revenue of $5,000 and a total cost of $750. One month, the firm owner decides to invest in more equipment. This extra equipment allows the company to build a sixth fence per month, but raises total cost to $825. Assuming the firm charges the same price for the sixth fence as it did for each of the other five, what is the change in total profit that results from increasing output? Find the value of MP! Need help! Do I have corona? \_()_/ You performed tayammum and worn your socks and you found water before next prayer. Can you wipe over your socks while performing ablution ? Why or Why not? Which situations can be represented by the equation 3+15=288? Select all that apply.Nate buys a phone that costs $288. He is able to put $15 down and then pay the remaining balance in 3 equal payments. What is x, the amount of one payment?Nate upgrades his phone. He makes 3 equal payments of $288 each and pays an additional $15 in taxes. What is x, the total cost of the phone?Nate bought his 3 children identical phones and chargers. He spent a total of $288. If each charger was $15, what is x, the cost of one phone?Nate bought his 3 children identical phones and chargers. He spent a total of $288. If each charger was $5, what is x, the cost of one phone?Nate bought his 3 children identical phones totaling $288. Nate used a coupon for $15 off total purchase. What is x, the discounted price of one phone? SOMEONE PLEASE HELP ME ASAP PLEASE!!!